1 Mark Type (Real Numbers)

You might also like

Download as pdf or txt
Download as pdf or txt
You are on page 1of 21

CHAPTER-1

REAL NUMBERS
01 MARK TYPE QUESTIONS
Q. NO QUESTION MARK
1. In my colony, number of males and females living are 32 and 36 respectively. What is the 1
minimum number of sweets to be purchased on 77th Independence day 2023 so that
everyone will get equal number of sweets?
a) 272
b) 288
c) 274
d) 144
2. In the following figure, which relationship is true? 1

a) y = x b) y > x c) z < y d) z > y


3. If x is a prime number and a be any positive integer such that a2 is a multiple of x, the 1
which of the following statement is true?
a) x can divide a2
b) x may or may not divide a
c) x divides a
d) Both (a) and (b) are correct
4. If a and b are any two integers and b = 3, then a can be expressed as 1
a) 3q, 3q+ 1, 3q + 2
b) 3q
c) none of the above
d) 3q+ 1
* Here q is some integer
5. 1

In a Block, the villagers from three Panchayats with strength 280, 504 and 672 respectively
want to go on a picnic. Buses are to be hired to take these villagers to the picnic. What is
the maximum number of villagers who can sit in a bus if each bus takes equal number of
villagers?
a) 72
b) 56
c) 77
d) 85
6. If X, Y and Z are three are positive integers such that X = m3n2p , Y = mn3p2 and Z = 1
m2n2p2 , the find the HCF(X,Y,Z).
7. The HCF of y and 153 is 51, where y is a natural number. Which of these can be true for 1
some values of y ?
i) y is a multiple of 51
ii) y is a multiple of 153
iii) y is a an even number
iv) y is a an odd number
a) only (ii) and (iii) b) only (i), (ii) and (iii)
c) only (i), (iii) and (iv) d) all (i), (ii),(iii) and (iv)
8. The LCM of the two numbers is 202 X 3. Which of the following cannot be their HCF? 1
Justify your reason.

a) 200
b) 400
c) 600
d) 500

9. If 3 is the least prime factor of number a and 7 is the least prime factor of number b, then 1
the least prime factor of a + b is

a) 5
b) 2
c) 3
d) 10
10. If n is a natural number, then which can always divide 92n – 42n ? 1
11. Aadya has 143 stamps; she gives away 11 stamps and divides the remaining 1
equally into groups. MARK
Sumit has 220 stamps; he gives away 11 stamps and divides the remaining
equally into groups.
They end up with the same number of groups.

What is the number of groups?

12. Aadya has 143 stamps; she gives away 11 stamps and divides the remaining 1
equally into groups. MARK
Sumit has 220 stamps; he gives away 11 stamps and divides the remaining
equally into groups.
They end up with the same number of groups.

What is the number of stamps in Adya’s and Sumit’s groups?


13. To enhance the reading skills of grade X students,the school nominates 1MARK
you and two of your friends to set up a class library.There are two sections-
section A and section B of grade X. There are 32 students in section A and 36
students in section B.

What is the minimum number of books you will acquire for the class
library, so that they can be distributed equally among students of Section A
or Section B?

a) 144 b) 128 c) 288 d) 272


14. To enhance the reading skills of grade X students,the school nominates 1MARK
you and two of your friends to set up a class library.There are two sections-
section A and section B of grade X. There are 32 students in section A and 36
students in section B.

If the product of two positive integers is equal to the product of their HCF
and LCM is true then, the HCF (32 , 36) is
a) 2 b) 4 c) 6 d) 8
15. 1MARK
To enhance the reading skills of grade X students,the school nominates
you and two of your friends to set up a class library.There are two sections-
section A and section B of grade X. There are 32 students in section A and 36
students in section B.

36 can be expressed as a product of its primes as


a) 2² × 3²
b) 2¹ × 3³
c) 2³× 3¹
d) 2⁰ × 3⁰
16. A seminar is being conducted by an Educational Organisation, where the 1MARK
participants will be educators of different subjects. The number of
participants in Hindi, English and Mathematics are 60, 84 and 108
respectively.

In each room the same number of participants are to be seated and all of
them being in the same subject, hence maximum number participants that
can accommodated in each room are

a) 14 b) 12 c) 16 d) 18

17. 1
A seminar is being conducted by an Educational Organisation, where the MARK
participants will be educators of different subjects. The number of
participants in Hindi, English and Mathematics are 60, 84 and 108
respectively.

What is the minimum number of rooms required during the event?

a) 11 b) 31 c) 41 d) 21

18. A seminar is being conducted by an Educational Organisation, where the 1MARK


participants will be educators of different subjects. The number of
participants in Hindi, English and Mathematics are 60, 84 and 108
respectively.
The LCM of 60, 84 and 108 is

a) 3780 b) 3680 c) 4780 d) 4680


19. A Mathematics Exhibition is being conducted in your School and one of 1
your friends is making a model of a factor tree. He has some difficulty and MARK
asks for your help in completing a quiz for the audience.
Observe the following factor tree and answer the following:

What will be the value of x?

a) 15005 b) 13915 c) 56920 d) 17429


20. A Mathematics Exhibition is being conducted in your School and one of 1MARK
your friends is making a model of a factor tree. He has some difficulty and
asks for your help in completing a quiz for the audience.
Observe the following factor tree and answer the following:

According to Fundamental Theorem of Arithmetic 13915 is a

a) Composite numberb) Prime numberc) Neither prime nor composited) Even


number
21. Ishani was teaching counting from 1 to 10 to her brother who is a Kindergarten student. 1
Suddenly one question came to her mind. Are there any numbers which are divisible by all
the numbers from 1 to 10? She found that least number that is divisible by all the
numbers from 1 to 10 which is
a) 10 b) 100 c)504 d) 2520
22. If 3 is the least prime factor of p, and 7 is the least prime factor of q, then the least prime 1
factor of (p+q) is:
a)11 b)2 c)5 d)10

23. n2 -1 is divisible by 8 if n is 1
a)an integer b) a natural number
c) an odd integer d) an even integer
24. L.C.M. of two co-prime numbers is always 1
(a) product of numbers (b) sum of numbers
(c) difference of numbers (d)none

25. The smallest irrational number which should be added to 1


4 − √5 to get a rational number is:
a ) √4 - 5 b) -√5 c) -4 +√5 d) √5

26. In the following questions, a statement of assertion (A) is followed by a statement of 1


Reason (R). Choose the correct answer out of the following choices.
Assertion (A): If LCM of two numbers is 2475 and their product is 12375, then their HCF is
5.
Reason (R): HCF (a, b) × LCM (a, b) = a × b.
(a) Both A and R are true and R is the correct explanation of A.
(b) Both A and R are true but R is not the correct explanation of A. (c) A is true but R is
false.
(d) A is false but R is true.
27. If the HCF of 408 and 1032 is expressible in the form 1
1032p – 408 × 5 find p.
a) 4 b) 2 c) 3 d)1
28. The largest number which divides 70 and 138, leaving remainders 5 and 8, respectively, is 1
(a) 13 (b) 65 (c) 875 (d) 1750

29. In the following questions, a statement of assertion (A) is followed by a statement of 1


Reason (R). Choose the correct answer out of the following choices.
Assertion (A): For no value of n, where n is a natural number, the number 6n ends with
the digit zero.
Reason (R): For a number to end with digit zero, its prime factors should have 2 and 5.
(a) Both A and R are true and R is the correct explanation of A.
(b) Both A and R are true but R is not the correct explanation of A.
(c) A is true but R is false.
(d) A is false but R is true
30. If two positive integers a and b are written as a = x3y2 and b = xy3 ; x, y are prime numbers, then 1
HCF (a, b) is
(a) xy (b) xy2 (c) x3y3 (d) x2y2

31. LCM of 550 and 121 is :


a. 11 b. 15
c. 10 d. 20
32. The largest number that can divide 615 and 963 leaving remainder 6 in each
case is :
a. 57
b. 87
c. 54
d. 84

33. Greatest Common Divisor of two numbers is 8 while their Least Common
Multiple is 144. Find the other number if one number is 16.

a. 108
b. 96
c. 72
d. 36
34. For any integer n, the odd integer can be represented as:

a. n
b. n + 1
c. 2n
d. 2n + 1
35. Which of the following option is not irrational?

a. (3 + √7)
b. (3 – √7)
c. (3 + √7) (3 – √7)
d. 3√7
36. If p and q are integers and is represented in the form of p/q, then it is a:

a. Whole number
b. Rational number
c. Natural number
d. Even number
37. The sum of two irrational numbers is always

a. rational
b. irrational
c. rational or irrational
d. not determined
38. Consider two positive integers a and b, what is HCF (a, b) × LCM (a, b) =

a. a + b
b. (a × b)/2
c. a/b
d. a × b
39. Which of the following represent √400 in the given diagram?
a. B
b. C
c. D
d. A
40. The prime factorisation of 96 is

(a) 2×2×2×2×2 × 3

(b) 2×4×5

(c) 2×6×3 × 3

(d) 2×3×4 × 3
41. Given that HCF(a,b)=1 and LCM(a,b)=ab, what can you say about a and b 1
(a) a and b are co-primes (c) a and b are prime numbers
(b) a and b are composite numbers (d) none of the above
42. 1
Reshma has decided to start a crèche with 10 kids of 3yrs age group and 7
kids of 4yrs age group. She needs to buy toys for the kids. What will be
the minimum number of toys she should purchase so that all the toys can
be equally distributed among all the kids.
(a) 85 (c) 77
(b) 70 (d) 10
43. A certain number n is divisible by 21, 28 and 49. What is the smallest value of 1
n?
(a) 500 (c) 588
(b) 508 (d) 580

44. If x and y are positive integers such that x is an even prime and y is even 1
composite, then which of the following is true?
(a) y - x cannot be an even integer (c) xy cannot be an even integer
(b) (x+y)/x cannot be an odd integer (d) x + y is an even integer
45. If p1 and p2 are two prime numbers one odd and another even such that p1> 1
p2 , then p12 - p22 is
(a) an even number (c) an odd number
(b) an odd prime number (d) Both (b) & (c)
46. If p is a prime number then, what is the LCM of p, p2, p3, p4, p5, p6 is 1
(a) p (c) p6
(b) p24 (d) p2
47. Jasmin is making a dress on her own. She has two pieces of lace. The first 1
piece is 45 centimeters long and the second one is 90 centimeters long. She
wants to cut the lace equally in length and as long as possible. How long
should Jasmin cut each lace?
(a) 90 (c) 45
(b) 5 (d) 10
48. Given that HCF(a,b) = √4 and LCM(a,b) = b, what can you say about b 1
(a) b = 5 (c) b = 1
(b) b is an odd number (d) b is an even number
49. The product of a non zero number and an irrational number is always 1
(a) Irrational Number (c) Even Number
(b) Rational Number (d) Odd Number

50. Which of the follow is not irrational 1


a) (3 + √7) (c) (3 - √7)
(b) (3 + √7)(3 - √7) (d) 3√7

51. if p and q are two distinct prime numbers, then their HCF is 1
(a) 2
(b) 0
(c) Either 1 or two
(d) 1
52. The smallest number divisible by all natural numbers between 1 and 10 (both 1
inclusive) is
(a) 2020
(b) 2520
(c) 1010
(d) 5040
53. The LCM of the smallest two digit number and the smallest composite 1
number is
(a) 12
(b) 20
(c) 4
(d) 44
54. If the HCF of 85 and 153 is expressible in the form of 85n-153, then the value 1
of n is

(a) 3
(b) 2
(c) 4
(d) 1
55. If the LCM of two numbers is 3600, then which of the following number 1
cannot be their HCF?

(a) 600
(b) 500
(c) 400
(d) 150
56. The sum of the HCF and LCM of 12, 21, 15 is 1

(a) 423
(b) 420
(c) 417
( d) 140
57. Given that HCF(2520, 6600)=40 and LCM(2520, 6600)=252 x K, then the 1
value of K is

(a) 1650
(b) 1600
(c) 165

(d)1625
58. 11 1
The decimal representation of 3 will
2 ×5
(a) Terminate after 1 decimal place
(b) Terminate after 2 decimal place
(c) Terminate after 3 decimal place
(d)Not terminate
59. If two positive integers are a and b are written as 𝑎 = 𝑝3 × 𝑞 4 and 𝑏 = 1
𝑝2 × 𝑞 3 , where p and q are prime numbers, such that HCF(a, b) ≠ 𝑝𝑟 × 𝑞 𝑠 ,
then (m+n)(r+s) equal to

(a) 15
(b) 30
(c) 35
(d) 72
60. The smallest number which when divided by 17, 23 and 29 leaves a remainder 1
11 in each case is

(a) 493
(b) 11350
(c) 11339
(d) 667
61. What is the HCF of smallest prime number and the smallest composite number ? 1
a) 2 b) 1 c) 5 d) 1.5

62. The HCF and LCM of 12,21,15 respectively are: 1


a) 3,140 b) 12,240 c) 3,420 d) 420, 3
63. There are 576 boys and 448 girls in a school that are to be divided into equal size sections of 1
either boys or girls alone. Find the minimum total number of sections thus formed.
a) 22 b)16 c)36 d)21

64. Three bulbs red, green, and yellow flash at intervals of 80,90, and 110 seconds. all three flashes 1
together at 8:00 am. when will they flash altogether again?
a) 9 am b)9:12 am c)10:12 am d) 12:10 pm
65. If the HCF of 65 and 117 is expressible in the form 65m – 117, then the value of m is 1
(a) 4 (b) 2 (c) 1 (d) 3
66. For any two positive integers a and b, HCF (a, b) × LCM (a, b) = 1
(a) 1 (b) (a × b)/2 (c) a/b (d) a × b
67. If two positive integers m and n are expressible in the form m = pq3 and n = p3q2, 1
where p, q are prime numbers, then HCF (m, n) =
(a) pq (b) pq2 (c) p3q2 (d) p2q2
68. Assertion (A) :-LCM of 13 and 61 is 793 1
Reason (R):- If a and b are prime ,then LCM(a,b) = a X b
a) both Assertion and reason are correct and reason is correct explanation for assertion
b) both Assertion and reason are correct but reason is not correct explanation for Assertion
c) Assertion is correct but reason is false
d) Assertion is false but reason is true
69. Assertion (A) :-HCF of 105 and 225 is 15 and LCM( 105,225)=225 x K, then the value of 1
K is 7.
Reason (R):- If any two positive number a and b, HCF (a,b) xLCM(a,b) = a x b
a) both Assertion and reason are correct and reason is correct explanation for assertion
b) both Assertion and reason are correct but reason is not correct explanation for Assertion
c) Assertion is correct but reason is false
d) Assertion is false but reason is true
70. Assertion (A) :-√3 is an irrational number. 1
Reason (R):- If p is prime then √p is an irrational number
a) both Assertion and reason are correct and reason is correct explanation for assertion
b) both Assertion and reason are correct but reason is not correct explanation for Assertion
c) Assertion is correct but reason is false
d) Assertion is false but reason is true
71. If 98 is expressed as a product of its primes. What will be the sum of the exponent of its 1
prime?
(a) 1 (b) 2 (c) 3 (d) 4
72. The least number which is divisible by all the first Five Natural numbers is: 1
(a) 60 (b) 90 (c) 120 (d) 80
73. If two positive integers a and b can be expressed as a = 𝑝2 𝑞, and b = 𝑝3 𝑞 2 ; where p, q 1
being prime numbers, then HCF (a, b) is equal to
(a) pq (b) 𝑝2 𝑞 2 (c) 𝑝3 𝑞 2 (d) 𝑝2 𝑞
74. If P is a prime number then, what is the LCM of p ,𝑝2 , 𝑝3 ? 1
(a) p (b)pxp (c) 0 (d) 𝑝3
75. The HCF of two consecutive even number is: 1
(a) 0(b) 1(c) 2 (d) 4
76. The LCM of smallest two digits composite number and smallest composite number is: 1
(a) 12 (b) 4 (c) 20 (d) 44
77. If p and q are two consecutive prime numbers greater than 2, what can you say about 𝑝2 - 1
𝑞2 ?
a) It's even b) It's odd
c) It's a multiple of 3 d) It's a prime number
78. Which of the following numbers is both rational and an integer? 1
1
a) √2 b) 3 c) 0.75 d) 4
79. Which of the following is a Rational number? 1
1 2√2
e. (b) (c) √2 (d) √2 + 2
√2 √2
80. A Muscle Gym has bought 63 treadmills and 108 elliptical machines. The gym divides 1
them into several identical sets of treadmills and elliptical machines for its branches located
throughout the city, with no exercise equipment left over. What is the greatest number of
branches the gym can have in the city?
(a) 7 (b) 9 (c) 108 (d) 63
81. If n is any natural number , then 6 − 5𝑛 always ends with
𝑛
1
(a) 1 (b) 3 (c) (d) 7
82. The HCF of x and 36 is 12 , where x is a natural number which of these can be true for 1
some values of x ?
(i) x is a multiple of 12 . (ii) x is a multiple of 36 .
(iii) x is an even number . (iv) x is an odd number .
(a) only (ii) and (iii) (c) only (i) , (ii) and (iii)
(b) only (i) and (iii) (d) All (i) , (ii) , (iii) and (iv)
83. If the LCM of x and 169 is 338 and the HCF of x and 169 is 13 then x = …. 1
(a) 26 (b) 13 (c) 169 (d) 1
84. The exponent of 3 in the Prime factorization of 324 is 1
(a) 5 (b) 2 (c) 4 (d) 3
85. The sum of the exponents of the Prime factors in the prime factorization of 196 is 1
(a) 1 (b) 2 (c) 4 (d) 6
86. If n is a natural number , then the number of consecutive zeroes in 6𝑛 , is 1
(a) 2 (b) 0 (c) 3 (d) 1
87. Three bells toll at intervals of 6, 9 , 12 minutes respectively . If they start tolling together . 1
After how many minutes will they next toll together .
(a) 20 minutes (c) 36 minutes
(b) 24 (d) 12 minutes
88. Which of the following is not an irrational number .. 1
(a ) 2√5 (b) √5 + 2√5 (c) ( √10+3 ) - √10 (d) √3 + 2
89. If two positive integers x and y are written as x = a³b² and y=a²b where a,b are prime 1
numbers then HCF (x,y) = …
(a) a³b (b) ab (c) a²b (d) a³b²
90. If a=2³ , b= 2 x 3 x 5 1
𝑛
c= 3 x 5 and LCM (a,b,c) .
= 2³ x 3² x 5 , then n =…………
(a) 1 (b) 2 (c) 3 (d) 4
91. Two representations of real numbers are shown below.
Which one is correct?

92. If p and q are positive integers such that p = ab2 and q= a2b, where a , b are prime numbers,
1
then the LCM (p, q) is
a) ab
b) a2b2
c) a3b2
d) a3b3
93. Let p & q be two natural numbers such that p > q. When p is divided by q, the remainder is
r.
i) r CANNOT be (p-q)
ii) r CAN either be q or (p-q)
iii) r is DEFINITELY less than q.
1
Which of the above statements is/are true?
1. only (ii)
2. only (iii)
3. only (i) & (iii)
4. (cannot be determined without knowing the values of p,q,r )
94. The prime factorization of a prime number is the number itself.
1
How many factors and prime numbers does the square of a prime number have ?
95. Find the HCF of 52 and 117 and express it in form
1
52x + 117y.
96. The HCF of k and 93 is 31, where k is a natural number.
Which of these CAN be true for SOME VALUES of k ?
i) k is a multiple of 31.
ii) k is a multiple of 93.
iii) k is an even number.
iv) k is an odd number. 1

1. Only (ii) & (iii)


2. Only (i), (ii) & (iii)
3. Only (i), (iii) & (iv)
4. All (i), (ii), (iii) & (iv)
97. The prime factorisation of 13915 is
(a) 5 × 113 × 132
(b) 5 × 113 × 232 1
(c) 5 × 112 × 23
(d) 5 × 112 × 132
98. Find the least number that is divisible by all numbers between 1 to 10. ( Both Inclusive ) 1
99. Find the greatest number of 6 digits exactly divisible by 24, 15 & 36. 1
100. If a & b are two prime numbers, write their LCM. 1
101. If p1 and p2 are two odd prime numbers such that p1> p2, then p12 – p22 1
is
(a) An even number
(b) An odd prime number
(c) An odd number
(D) A prime number
102. If two positive integers a and b are written as a=x3y2 and b=xy3 ,x,y are prime 1
numbers ,then HCF (a,b ) is
(a) xy
(b) xy2
(c) x3y3
(d) x2y2
103. If n2-1 is divisible by 8 then n is 1
(a) An integer
(b) A natural number
(c) An odd integer
(d) An even integer
104. f the HCF of 408 and 1032 is expressible in the form 1032 × 2 + 408 × p, then the value of 1
p is
-3
(a)
(b) -4
(c) -5
(d) -6
105. If A= 2n +13 and B =n+7 , where n is a natural number, then HCF of A and B is 1
(a) 2
(b) 3
(c) 4
(d) 1
106. The product of three consecutive positive integers is divisible by 1
(a) 4
(b) 6
(c) no common factor
(d) only 1
107. For some integer p, every even integer is of the form 1
(a) 2p + 1
(b) 2p
(c) p + 1
(d) p
108. If n is a natural number ,then 92n – 42n is always divisible by 1
(a) 5
(b) 13
(c) Both 5 and 13
(d) None of these
109. 1

What is the minimum number of books you will acquire for the class library,
so that they can be distributed equally among students of Section A or
Section B?

a) 144 b) 128 c) 288 d) 272


110. If the product of two positive integers is equal to the product of their HCF 1
and LCM is true then, the HCF (32 , 36) is
a) 2

b) 4

c) 6

d) 8
111. 36 can be expressed as a product of its primes as 1
a) 22 × 32
b) 21 × 33
c) 23 × 31
d) 20 × 30
112. 7 × 11 × 13 × 15 + 15 is a 1
a) Prime number

b) Composite number

c) Neither prime nor composite

d) None of the above


113. 1

There are 312, 260 and 156 students in class X, XI and XII
respectively. Buses are to be hired to take these students to a
picnic. Find the maximum number of students who can sit in a
bus if each bus takes equal number of students
(a) 52
(b) 56
(c) 48
(d) 63
114. Three farmers have 490 kg, 588 kg and 882 kg of wheat 1
respectively. Find the maximum capacity of a bag so that the
wheat can be packed in exact number of bags.
(a) 98 kg
(b) 290 kg
(c) 200 kg
(d) 350 kg
115. 1

There is a circular path around a sports field. Priya takes 18 minutes to


drive one round of the field. Harish takes 12 minutes. Suppose they
both start at the same point and at the same time and go in the same
direction. After how many minutes will they meet?
(a) 36 minutes
(b) 18 minutes
(c) 6 minutes
(d) They will not meet
116. If HCF (26, 169) = 13, then LCM (26, 169) = 1
(a) 26
(b) 52
(c) 338
(d) 13
117. If two positive integers m and n are expressible in the form m = 1
pq3 and n = p3q2 where p, q are
prime numbers, then HCF (m, n) =
A. pq
B. pq2
C. p3q3
D. p2q3
118. If n = 23 × 34 × 54 × 7, then the number of consecutive zeros in n, 1
where n is a natural number, is
(a) 2
(b) 3
(c) 4
(d) 7
119. Which of the following is an irrational number? 1
A) 0.131131113...

B) 0.25

C) 0.4

D) 0.125

ANSWERS:
Q. NO ANSWER MARKS
1. b) 288 1
2. d) z > y 1
3. d) Both (a) and (b) are correct 1
4. a) 3q, 3q+ 1, 3q + 2 1
5. b) 56 1
6. HCF(X,Y,Z) = mn2p 1
7. c) only (i), (iii) and (iv) 1
8. d) 500 1
9. b) 2 1
10. Both 5 and 13 1
11. 11
12. Adya’s groups have 12 stamps each and , Sumit’s groups have 19 stamps each 19, 1212, 19
12 stamps and 19 stamps
13. c) 288
14. b) 4
15. a) 2² × 3²
16. b) 12
17. d) 21
18. a) 3780
19. b) 13915
20. a) Composite number
21. (d)2520 1
22. (b) 2 1
23. c) an odd integer 1
24. a) product of numbers 1
25. d) √5 1
26. (a) Both A and R are true and R is the correct explanation of A. 1
27. b) 2 1
28. b) 65 1
29. (a) Both A and R are true and R is the correct explanation of A 1
30. b) xy2 1
31. a 1
32. b 1
33. c 1
34. d 1
35. c 1
36. b 1
37. c 1
38. d 1
39. d 1
40. a 1
41. Let a = 2, b = 3 1
HCF(2,3) = 1, LCM(2,3) = 6,
Therefore 2 and 3 are co-primes
Ans - (a)
42. LCM(10,7) = 70 1
Ans - (b)
43. LCM(21,28,49) = 588 1
Ans - (c)
44. Let y = 12, x = 2 1
y - x = 12 - 2 (even)
xy = 12 x 2 = 24 (even)
(x+y)/x = (12+2)/2 = 14/2 = 7(odd)
x + y = 12 + 2 = 14 (even)
Ans - (d)
45. Let p1 = 3, p2 = 2 1
p12 = 9, p22 = 4
p12 - p22 = 5 (odd prime)

Let p1 = 5, p2 = 2
p12 = 25, p22 = 4
p12 - p22 = 25 - 4 = 21 (odd)

Ans - (d) Both (b) and (c)


46. LCM(p, p2,p3,p4,p5,p6) = Highest power of common factor 1
Ans - (c)
47. LCM (45,90) = 45 1
Ans - (c)
48. Let a = 2 and b = 6 1
HCF (2,6) = 2 and LCM (2,6) = 6

Let a = 2, b = 8
HCF (2,8) = 2 and LCM (2,8) = 8

Let a = 2, b = 5
HCF (2,5) = 1 and LCM (2,5) = 10
Ans - (d)
49. 9 x √3 = 9√3 (Irrational Number) 1
Ans - (a)
50. (3)2 - (√7)2 = 9 - 7 = 2 (Rational) 1
Ans - (b)
51. d 1
52. b 1
53. b 1
54. b 1
55. b 1
56. a 1
57. a 1
58. c 1
59. c 1
60. b 1
61. a) 2 1
62. c) 3,420 1
63. b) 16 1
64. c) 10:12 am 1
65. b) 2 1
66. (d) a × b 1
67. (b) pq2 1
68. a) both Assertion and reason are correct and reason is correct explanation for 1
assertion
69. a) both Assertion and reason are correct and reason is correct explanation for 1
assertion
70. a) both Assertion and reason are correct and reason is correct explanation for 1
assertion
71. (c) 3 1
72. (a)60 1
73. (d) 𝑝2 𝑞 1
74. (d) 𝑝3 1
75. (c) 2 1
76. (c) 20 1
77. (a) It's even 1
78. (d) 4 1
79. (b)
2√2 1
√2
80. (b) 9 1
81. (a) 1 1
82. (b) only (i) and (iii) 1
83. (a) 26 1
84. (c) 4 1
85. (c) 4 1
86. (b) 0 1
87. (b) 36 minutes 1
88. (c) ( √10+3 ) - √10 1
89. (c) a²b 1
90. (b) 2 1
91. Representation 1. 1
92. b) a2b2 1
93. 2. only (iii) 1
94. For the square of a prime number:
Number of Factors = 3 1
Number of Prime Factors = 1
95. The HCF of 52 and 117 is 13, and it can be expressed in the form of 52x and 117y as
1
52(5) + 117(-4) = 13.
96. 3.Only (i), (iii) & (iv) 1
97. (c) 5 × 112 × 23 1
98. The required number is the LCM of 1,2,3,4,5,6,7,8,9,10. 1
LCM = 2 x 2 x 3 x 2 x 3 x 5 x 7 = 2520
99. LCM of 24, 15, 36
LCM = 3 x 2 x 2 x 2 x 3 x 5 = 360
Now the greatest 6 – digit number is 999999
1
Divide 999999 by 360
Hence, Q = 2777, R = 279
Therefore, the required number is 999999 – 279 = 999720
100. LCM = a x b 1
101. a 1
102. b 1
103. c 1
104. c 1
105. d 1
106. b 1
107. b 1
108. c 1
109. b 1
110. b 1
111. a 1
112. b 1
113. a 1
114. a 1
115. a 1
116. c 1
117. b 1
118. b 1
119. a 1

You might also like